Find the measure of EB

Find The Measure Of EB

Answers

Answer 1

The measure of angle subtended by the arc EB is 96 ⁰.

What is the measure of arc angle EB?

The measure of angle subtended by the arc EB is calculated by applying the following formula.

Based on the angle of intersecting chord theorem, the theory states that, the angle formed by the intersection of two chords at the circumference of a circle is equal to half of the difference between the arc angles of the two chords.

We will have the following equation;

m∠ECB = ¹/₂( 7x + 6 - (4x + 16))

25 x 2 = 7x + 6 - 4x - 16

50 = 3x - 10

60 = 3x

x = 60/3

x = 20

The measure of arc angle EB is calculated as follows;

m∠EB = 4x + 16

m∠EB = 4(20) + 16

m∠EB = 96 ⁰

Learn more about chord angles here: brainly.com/question/23732231

#SPJ1


Related Questions

What is the value of X? (Please help, used 50 points!)

Answers

Answer:

x=7

(x)(x+1)=(4)(4+10)

Step-by-step explanation:

Expanding the right-hand side of the equation, we get:

(x)(x+1) = (4)(14)

Simplifying, we get:

x^2 + x = 56

Moving all terms to the left-hand side, we get:

x^2 + x - 56 = 0

Now we can factor this quadratic equation:

(x + 8)(x - 7) = 0

So the solutions are x = -8 or x = 7.

Answer: X=7

Step-by-step explanation:

To Solve this problem we have to remember a formula regarding secants in circles it states (A+B)*B=(C+D)*D, I have attached said Theorem. This is Exactly what your problem is asking.

So First

B = 4

A = 10

D = x

C = 1

(A+B)*B=(C+D)*D

(10+4)*4=(1+x)*x

14*4=x²+x

-x²-x+56=0

Using Quadratic Formula

X = 7, X = -8

Since you cant have - side length X= -8 is Extraneous

Thus Final solution is X = 7

What is the domain of g(x)=log(7x−10)?

Answers

The domain of g(x) = log (7x - 10) is all real numbers greater than 10/7.

Set-builder notation: x > 10/7. Interval notation: (10/7,∞)

What is the domain of the function?

Given the function in the question:

g(x) = log( 7x - 10 )

For the function g(x) = log (7x - 10), the domain consists of all the possible values of x that can be plugged into the function without resulting in an undefined expression.

Here, the logarithm (in this case, (7x - 10) must be greater than zero, since the logarithm of zero or a negative number is undefined in the real numbers.

So we set the argument of the logarithm to be greater than zero, and solve for x:

7x - 10 > 0

Solve for x

7x - 10 + 10 > 0 + 10

7x > 10

Dividing both sides by 7:

7x/7 > 10/7

x > 10/7

Therefore, the domain of g(x) = log (7x - 10) is x > 10/7

Learn more about domain here: https://brainly.com/question/30133157

#SPJ1

Find the graphed inequality below

Answers

The inequality represent the given graph is y≤1.4x+2.5.

From the graph, the line passes through (0, 2.5) and (3.5, 0).

Here, slope (m) = (3.5-0)/(2.5-0)

= 3.5/2.5

= 1.4

Substitute m=1.4 and (x, y)=(0, 2.5) in y=mx+c, we get

2.5=1.4(0)+c

c=2.5

So, the equation of a line is y=1.4x+2.5

So, the inequality is y≤1.4x+2.5

Therefore, the inequality represent the given graph is y≤1.4x+2.5.

To learn more about the inequalities visit:

https://brainly.com/question/20383699.

#SPJ1

Which expression is not equivalent to 2/3×4

Answers

The only expression that is not equivalent to the given fraction expression is: D (2 x 1/3) + (4 x 1/3)

How to multiply fractions?

The correct procedure that we will use to multiply fractions is:

- find a common denominator

- multiply the numerators

- multiply the denominators

- Simplify if necessary.

- Add the numerators and add the denominators

Looking at the options and comparing with the given fraction multiplication problem 2/3 * 4, we see that only option D is not equivalent to it because:

(2 x 1/3) + (4 x 1/3) = 6/3 = 2

Read more about Fraction Multiplication at: https://brainly.com/question/7335118

#SPJ1

Complete question is:

Which expression is NOT equivalent to 2/3 x 4

A (2x4) /3

B 1/3 x (2x4)

C (4 x 1/3) x 2

D (2 x 1/3) + (4 x 1/3)

Choose one of the topics below for your paragraph.
1. Write a paragraph about a trend that interests you and why it is popular. Give sentences
(Paragraph)
● For example, consider a trend in cars, food, or technology. Give sentences (Paragraph)
2. Write a paragraph about a popular TV program. Give sentences (Paragraph)
Who usually watches the program? Give sentences (Paragraph)
Why is it popular? Give sentences (Paragraph)

Answers

Answer:

1. A trend that gives me interest is the 1, 2 buckle my shoes 3, 4 buckle some more- The reason is why does it even exist and why did it get so popular is such a fast amount of time, and the way people get distracted on technology which we are using all over the world and it takes us away from the real world and makes us stay there.

2. Cocaine Bear it looks good and funny, I watched it with all my friends and they all enjoyed it with me.

Step-by-step explanation:

State the domain, the range, and the intervals over which the graphically defined function is increasing, decreasing, or constant.

Answers

The domain will be (-∞, ∞). The range will be (-1, 1]. The function will be increasing during the intervals (0, π/2) and (3π/2, 2π), and will be decreasing at the interval (π/2, 3π/2). It will be constant only at x=0 and not at any interval.

We are given a graph in the question and we have to determine its domain, range, and the intervals over which the defined function is increasing, decreasing, and constant. If we observe this graph, we can see that this is a sin(x) graph.

The domain of this graph will be from -[tex]\infty[/tex] to [tex]\infty[/tex] as we can see in the graph that it is a repeating pattern. The range for this graph will lie between -1 and 1 as it is a sin x graph. The sin x graph increases at the intervals  (0, π/2) and (3π/2, 2π) and decreases at the interval  (π/2, 3π/2). As we can see from the graph, it is constant only at a point where x = 0 and there is no interval at which this graph is constant.

To learn more about increasing and decreasing functions;

https://brainly.com/question/22554610

#SPJ1

Can anybody help me quick 21 points

Answers

Let the number of apartments be x. We know that the cost of the digits is $1 each, and that the total cost of the digits is $282. Therefore, we can set up the following equation:

1x + 1(x+1) + 1(x+2) + ... + 1(x+(n-1)) = 282

Simplifying this equation, we get:

nx + n(n-1)/2 = 282

We can solve for n using the quadratic formula:

n = (-1 ± sqrt(1 + 4(2*282)))/2

n = 21.06 or n = -20.06

Since the number of apartments must be a positive integer, we can round up to the nearest integer, giving us n = 22. Therefore, Tabitha will be numbering 22 apartments.

Extra: On Monday, the store sells 1s for half price, or $0.50. This means that the total cost of the digits would be:

1x + 0.5(x+1) + 0.5(x+2) + ... + 0.5(x+(n-1)) = 141

Simplifying this equation, we get:

nx + n(n-1)/4 = 141

Using the quadratic formula again, we get:

n = (-1 ± sqrt(1 + 4(4*141)))/2

n = 16.76 or n = -15.76

Rounding up to the nearest integer, we get n = 17. Therefore, if Tabitha had bought the digits on Monday, she could have saved 5 digits, or $5.

PLEASE HURRY

Let u = - 4i + 2j , v = 3i - j and w = - 6j

Find the specified scalar or vector

5u(3v - 4w)

Answers

The specified scalar or vector 5u(3v - 4w) is equal to -420i + 250j.

We can begin by distributing the scalar 5 to the vector 3v - 4w:

5u(3v - 4w) = 5u(3v) - 5u(4w)

Next, we can find the scalar multiples:

5u(3v) = 5(-4i + 2j)(3(3i - j)) = 5(-36i + 10j)

5u(4w) = 5(-4i + 2j)(4(-6j)) = 5(48i - 40j)

Now we can substitute these values back into the original expression:

5u(3v - 4w) = 5(-36i + 10j) - 5(48i - 40j)

Simplifying:

5u(3v - 4w) = -420i + 250j

Therefore, 5u(3v - 4w) = -420i + 250j.

To learn more about vector here:

https://brainly.com/question/29740341

#SPJ1

HELP ASAP!! 15 POINTS!!

Answers

Based on the information, we can infer that the correct graph/table is option D.

How to identify the correct table/chart?

To identify the correct chart or table, we must identify the information about the data that Emily uses each year. She uses 360GB each year, in this case, to identify the relationship we must divide the amount of data she uses by the number of months she has in a year:

360GB / 12 months = 30 GB/month

In accordance with the above, we must identify the table or graph that represents this relationship. In this case it would be table D because it shows an increasing relationship of 1 month with 30gb, 2 months with 60gb, 3 months with 90gb, 4 months with 120gb, and so on.

Learn more about relationship in: https://brainly.com/question/31248849

#SPJ1

there are 23 students in mr wilson's classroom. mr wilson wants to determaine the probabillities of male and female students playingon a school sports team. he surveyed his students and determained

Answers

The probability of boys and girls playing on school sport team is x/23 and y/23 respectively.

What is probability?

A probability is a number that reflects the chance or likelihood that a particular event will occur. Probabilities can be expressed as proportions that range from 0 to 1.

This means that the addition of probability of boys and probability of girls playing in a school tem will be 1. Represent the number of boys by x and the number of girls by y.

Probability = sample space /Total outcome

Since the total number of students is 23

Therefore, the probability of boys in the team = x/23 and,

Probability of girls = y/23

therefore x/23 +y/23 = 1

learn more about probability from

https://brainly.com/question/24756209

#SPJ1

Which of the following statements is true about the numbers below?
(i)
(the digits cycle through repeatedly)
(ii)
(the number of
inbetween the
increases by one each time)

Answers

The statement that is true about the numbers above include the following: C. (i) is rational and (ii) is rational.

What is a rational number?

In Mathematics and Geometry, there are six (6) common types of numbers and these include the following:

Irrational numbersIntegersReal numbersRational numbersNatural (counting) numbersWhole numbers

In Mathematics and Geometry, a rational number can be defined a type of number which comprises fractions, integers, terminating or repeating decimals such as the 2/3, 0.12345678901234567890.

What is an irrational number?

In Mathematics and Geometry, an irrational number can be defined a type of number which comprises non-terminating or non-repeating decimals such as the square root of 11 or √11.

Read more on rational number here: brainly.com/question/20400557

#SPJ1

Apply the sorted edges algorithm to the graph above. Give your answer as a list of vertices, starting and ending at vertex A. Example: ABCDEA

Answers

The sorted edges algorithm was applied to the pentagon prism graph ABCDEA with edge weights given. The final path is starting and ending at vertex A is  A-B-C-D-A-E.

The sorted edges algorithm starts by sorting the edges in ascending order of their weights. Then, starting from vertex A, the algorithm chooses the next smallest edge that connects to an unvisited vertex until all vertices have been visited.

Sort edges in ascending order

AD = 4, EA = 6, BC = 2, BD = 9, AB = 7, CE = 11, DE = 8, CA = 14, BE = 15, CD = 13

Starting at vertex A, select the next smallest edge that connects to an unvisited vertex AB = 7. Move to vertex B and select the next smallest edge that connects to an unvisited vertex BC = 2. Move to vertex C and select the next smallest edge that connects to an unvisited vertex CD = 13

Move to vertex D and select the next smallest edge that connects to an unvisited vertex AD = 4. Move to vertex A and select the next smallest edge that connects to an unvisited vertex EA = 6. Move to vertex E and all vertices have been visited. The final path is A-B-C-D-A-E.

To know more about vertex:

brainly.com/question/14815086

#SPJ1

Hey I was wondering if anyone can help me solve this and maybe graph it to.
The line described by y = 3/4x+5 is tangent to a circle at the point (0, 5). The line described by 3x + 4y = 38 is tangent to the same circle at the point (6, 5). Find the equation of the circle.

Answers

The equation of the circle is determined as (x + 3)²  + (y - 9)² = 25.

What is the equation of the circle?

The equation of the circle is calculated as follows;

For the equation;

y = 3/4x+5

Slope = 3/4,  the slope of the line perpendicular to the line = -4/3.

it is pass through the (0, 5).

The equation of the line: y - 5 = -4x/3

For the equation;

3x + 4y = 38

4y = -3x + 38

y = -3x/4 + 38

slope = -3/4, the slope of the line perpendicular to the line = 4/3

it is pass through the (6, 5)

The equation of the line: y - 5 = 4/3(x - 6)

Solve the two equations together to find the point of intersection;

(-4/3)x + 5 = (4/3)(x - 6) + 5

-4x/3 = 4x/3 - 8

8x/3 = -8

8x = -24

x = -3

y - 5 = -4x/3

y - 5 = -4(-3)/3

y - 5 = 4

y = 9

The center of the circle = (-3, 9)

The radius of the circle is calculated as;

r = √ (-3 -0)² + (9 - 5)²

r = √(9 + 16)

r = 5

The equation of the circle becomes;

(x - a )² + (y - b)² = r²

(x + 3)²  + (y - 9)² = 5²

(x + 3)²  + (y - 9)² = 25

Learn more about equation of circle here: https://brainly.com/question/1506955

#SPJ1

What is the product? -4x [8 -1 -5 g]

Answers

The resulting matrix after the product is given as follows:

[-32 4 20].

What happens when a matrix is multiplied by a constant?

When a matrix is multiplied by a constant, we have that every element in the matrix is multiplied by the constant. Hence, the dimension of the matrix remains constant.

The parameters for this problem are given as follows:

Constant of -4.Matrix [8 -1 -5].

Hence the products are:

-4 x 8 = -32.-4 x -1 = 4.-4 x -5 = 20.

Then the resulting matrix after the product is given as follows:

[-32 4 20].

More can be learned about multiplication of matrices at https://brainly.com/question/29202504

#SPJ1

A prisms base is a hexagon of a side length 10 The height of the prism is 14 Find the volume of the prism

Answers

Answer: The volume of the prism is 4200 cubic units.

Explanation:

The volume of a prism is given by the formula V = Bh, where B is the area of the base and h is the height. For a hexagonal base with a side length of 10, we can divide it into six equilateral triangles with side lengths of 10. Each equilateral triangle has an area of (1/2)bh, where b is the base and h is the height. Since the base of each equilateral triangle is also 10, we have:

Area of one equilateral triangle = (1/2)(10)(10√3/2) = 50√3

Area of the hexagonal base = 6 times the area of one equilateral triangle = 6(50√3) = 300√3

Now we can use the formula for the volume of a prism:

V = Bh = (300√3)(14) = 4200 cubic units.

Therefore, the volume of the prism is 4200 cubic units.

in the figure below, m JKM=106 degrees, m LKM=66, and KN bisects LKM. Find m JKN

Answers

The value of angle JKN is 73°.

Given that the m ∠JKM = 106 degrees, m ∠LKM = 66, and KN bisects LKM.

So we need to find the value of ∠JKN,

So, using the angles addition postulate,

∠LKM = ∠LKN + ∠MKN

Therefore,

∠LKN = 66/2 [definition of bisector]

∠LKN = 33°

Again, using the angles addition postulate,

∠JKM = ∠JKL + ∠LKM

∠JKL = 106 - 66

∠JKL = 40°

Therefore,

∠JKN = ∠JKL + ∠LKN

∠JKN = 40 + 33

∠JKN = 73°

Hence, the value of angle JKN is 73°.

Learn more about angles addition postulate, click;

https://brainly.com/question/29713158

#SPJ1

The percent of US citizens that use the internet on a daily basis is shown on the graph, where the year 2000 corresponds to x=0. Use the concepts of y-intercept and slope of the line to find the equation that best models the given data. Answer in slope intercept form.

Answers

An equation of the linear function represented by the graph in slope-intercept form is y = 3x + 60.

How to determine an equation of this line?

In Mathematics and Geometry, the point-slope form of a straight line can be calculated by using the following mathematical expression:

y - y₁ = m(x - x₁)

Where:

x and y represent the data points.m represent the slope.

First of all, we would determine the slope of this line;

Slope (m) = (y₂ - y₁)/(x₂ - x₁)

Slope (m) = (75 - 60)/(5 - 0)

Slope (m) = 15/5

Slope (m) = 3

At data point (0, 60) and a slope of 3, a linear equation for this line can be calculated by using the point-slope form as follows:

y - y₁ = m(x - x₁)

y - 60 = 3(x - 0)  

y = 3x + 60

Read more on point-slope here: brainly.com/question/24907633

#SPJ1

Missing information:

The question is incomplete and the complete question is shown in the attached picture.

According to Bureau of Labor Statistics26.0% of the total part-time workforce in the USwas between the ages of 25 and 34 during a recent monthA random sample of 90 part-time employees was selected during this quarter. Using the normal approximation to the binomial distribution, what is the probability that fewer than 18 people from this sample were between the ages of 25 and 342 (round standard deviation to 4 decimal places)

Answers

The probability that fewer than 18 people from this sample were between the ages of 25 and 34 is 0.1089 (or 10.89%).

Using the normal approximation to the binomial distribution, the mean (μ) of the sample is:

μ = np = 90 x 0.26 = 23.4

The standard deviation (σ) of the sample is:

σ = √(np(1-p)) = √(90 x 0.26 x 0.74) = 4.37 (rounded to 4 decimal places)

To find the probability that fewer than 18 people from this sample were between the ages of 25 and 34, we need to calculate the z-score:

z = (x - μ) ÷ σ = (18 - 23.4) ÷ 4.37 = -1.24

Using a standard normal table or calculator, we find that the probability of a z-score less than -1.24 is 0.1089.

0.1089 = 10.89%

To learn more about probability follow the link:

https://brainly.com/question/30034780

#SPJ1

Which statement is true about the system x - 3y = 5 and y = x - 7?
• (8, 1) is a solution to one equation but not the other one, so it is a solution to the system.
• (8, 1) is not a solution to either equation, so it is not a solution to the system.
• (8, 1) is a solution to one equation but not the other one, so it is not a solution to the system.
• (8, 1) is a solution to both equations, so it is a solution to the system.

Answers

The solution to the given equation is ( 8 , 1 )

Given data ,

To determine whether (8, 1) is a solution to the system, we need to check whether it satisfies both equations:

x - 3y = 5: 8 - 3(1) = 5, which is true

y = x - 7: 1 = 8 - 7, which is true

Since (8, 1) satisfies both equations, it is a solution to the system. Therefore, the correct answer is:

Hence , (8, 1) is a solution to both equations, so it is a solution to the system

To learn more about equations click :

https://brainly.com/question/19297665

#SPJ1

if an avocado is 1$ per pound how many is it for 9 pounds?

Answers

If an avocado costs $1 per pound, the cost of 9 pounds, using multiplication, is $9.

What is multiplication?

Multiplication is one of the four basic mathematical operations, including addition, subtraction, and division.

Multiplication involves the multiplicand, the multiplier, and the product.

The cost per pound of an avocado = $1

The total quantity considered = 9 pounds

Proportionately, the total cost of 9 pounds = $9 ($1 x 9)

Thus, based on multiplication operation, the total cost of 9 pounds is $9.00.

Learn more about mathematical operations at https://brainly.com/question/4721701.

#SPJ1


What’s the correct answer for problem 16???

Answers

The values of x and y of the given transverse lines are:

x = 44.25° and y = 7.25°

How to find the missing angle on the line?

We know that there are different classification of angles such as:

Corresponding angles

Alternate angles

Opposite angles

Supplementary angles

Complementary angles

Now, we also know that sum of angles on a straight line is 180 degrees. Thus:

5x - 9y - 16 = 140

5x - 9y = 156   -----(1)

We also know that alternate angles are defined as two angles, that are formed when a line crosses two other lines, that lie on opposite sides of the transversal line and on the opposite relative sides of the other lines. If the two lines crossed are parallel, then the alternate angles are equal.

Thus:

3x + y = 5x - 9y - 16

2x - 10y - 16 = 0

2x - 10y = 16  ----(2)

Solving simultaneously gives us:

x = 44.25° and y = 7.25°

Read more about Missing Angle at: https://brainly.com/question/28293784

#SPJ1

(x+3) (x-1)squared
????

Answers

Answer:

x^3 + x^2 - 5x + 3

Solve by using a system of equations.
A goldsmith has two gold alloys. The first alloy is 30% gold; the second alloy is 80% gold. How many grams of each should be mixed to produce 40 grams of an alloy that is 60% gold?

Answers

Answer:

Let x be the amount of the first alloy (30% gold) to be mixed and y be the amount of the second alloy (80% gold) to be mixed.

We can set up a system of two equations based on the information given:

x + y = 40 (total amount of alloy produced)

0.3x + 0.8y = 0.6(40) (amount of gold in the alloy produced)

Simplifying the second equation, we get:

0.3x + 0.8y = 24

Now we have a system of two equations:

x + y = 40

0.3x + 0.8y = 24

We can solve for x and y by using any method of solving systems of equations. Here, we will use the substitution method.

Solving the first equation for y, we get:

y = 40 - x

Substituting this expression for y into the second equation, we get:

0.3x + 0.8(40 - x) = 24

Simplifying and solving for x, we get:

0.3x + 32 - 0.8x = 24

-0.5x = -8

x = 16

So we need 16 grams of the first alloy and 24 grams of the second alloy to produce 40 grams of an alloy that is 60% gold.

what are the answers to these questions?

Answers

a) The function of h in terms of r is h=1400/(πr²).

b) The surface area of the can in terms of r is 2800/r+2πr².

c) The corresponding values of r and h that minimize the amount of required material are r≈14.04 cm and h≈2.98 cm.

a) The volume of a cylinder is given by

V = πr²h.

In this case, the volume of the can is

1400 cm³

So we have

1400 = πr²h.

Solving for h, we get

h = 1400/(πr²).

b) The surface area of a cylinder is given by

A = 2πrh+2πr².

Using the expression we obtained for h in part (a), we can substitute it in the equation and get

A = 2πr(1400/(πr²))+2πr²

= 2800/r+2πr².

c) To minimize the amount of required material, we need to find the value of r that minimizes the surface area.
To do this, we take the derivative of the surface area expression with respect to r and set it equal to zero:

dA/dr = -2800/r²+4πr = 0.

Solving for r, we get

r = √(700/π), which is approximately 14.04 cm.

Substituting this value in the expression for h that we obtained in part (a), we get

h = 1400/(π(14.04)²), which is approximately 2.98 cm.

To learn more about can click on,

https://brainly.com/question/31405743

#SPJ1

Find the missing side length.
Assume that all intersecting sides meet at right angles.
Be sure to include the correct unit in your answer.

Answers

Answer:

9ft

Step-by-step explanation:

From the left side, you know one length is 4ft abd the one below is 5ft, when you add them together its the answer of 9ft, which is the missing side.

it's 9ft

its 9ft because when you add 4 +5 you get 9. two rectangles a present in the diagram. the 4ft and 5ft are apart of the rectangle where the missing side is, adding those two together would give you 9 and in a rectangle sides facing each other are epual therefore the missing side is 9

Help me answer the question and please explain.

Answers

Thus, radius of the smaller cylinder is found as the 0.08 cm, for the given ratio of surface areas.

Explain about the cylindrical shape:

A cylinder's surface area provides information on how much space is present on the whole surface of the cylinder. If you were to use wrapping paper to wrap a can of Pringles as a birthday gift, then amount required would be equal towards the surface area of a Pringles can.

Let the surface area of smaller cylinder be s.

Let the  surface area of larger cylinder be S.

ratios of surface area:

s/S = 4 / 25

Radius of larger cylinder = 0.5 cm.

As,both cylinders are similar height will be same.

The formula for the surface area = 2*π*r*h

Put the values in ratio:

2*π*r*h / 2*π*R*h = 4 / 25

r / 0.5 = 4/25

cross multiplying:

r = 0.5*4 / 25

r = 0.08

Thus, radius of the smaller cylinder is found as the 0.08 cm.

Know more about the cylindrical shape:

https://brainly.com/question/25640064

#SPJ1

f(x)=x^3-3x^2+2x+2. find f(-x).

Answers

Answer:Therefore, f(-x) = -x^3 - 3x^2 - 2x + 2.

Step-by-step explanation:o find f(-x), we replace every instance of x in the function f(x) with -x:

f(-x) = (-x)^3 - 3(-x)^2 + 2(-x) + 2

Simplifying, we get:

f(-x) = -x^3 - 3x^2 - 2x + 2

A charity needs to report its typical donations received. The following is a list of the donations from one week. A histogram is provided to display the data.

10, 11, 35, 39, 40, 42, 42, 45, 49, 49, 51, 51, 52, 53, 53, 54, 56, 59

A graph titled Donations to Charity in Dollars. The x-axis is labeled 10 to 19, 20 to 29, 30 to 39, 40 to 49, and 50 to 59. The y-axis is labeled Frequency. There is a shaded bar up to 2 above 10 to 19, up to 2 above 30 to 39, up to 6 above 40 to 49, and up to 8 above 50 to 59. There is no shaded bar above 20 to 29.

Which measure of variability should the charity use to accurately represent the data? Explain your answer.

The range of 13 is the most accurate to use, since the data is skewed.
The IQR of 49 is the most accurate to use to show that they need more money.
The range of 49 is the most accurate to use to show that they have plenty of money.
The IQR of 13 is the most accurate to use, since the data is skewed.

Answers

Answer:

The IQR (interquartile range) of 13 is the most accurate measure of variability to use in this case, since the data is skewed and contains some outliers.

The IQR is a measure of variability that is less sensitive to extreme values than the range, and is calculated as the difference between the upper and lower quartiles (the 75th and 25th percentiles). It provides a measure of the spread of the middle 50% of the data, which is useful for understanding the typical range of donations received.

In this case, the IQR is calculated as follows:

- The median of the data is 51 (the value in the middle).

- The lower quartile (Q1) is the median of the lower half of the data, which is 42.

- The upper quartile (Q3) is the median of the upper half of the data, which is 54.

- The IQR is the difference between Q3 and Q1: IQR = Q3 - Q1 = 54 - 42 = 12.

So the IQR of 13 is a useful measure of variability to use for this data set, since it captures the spread of the middle 50% of the data while being less sensitive to the outliers at the higher end of the distribution.

-2√48 simplified is:
-8√3.
-4√3.
No solution.
None of these choices are correct.

Answers

Answer:

-8√3

Step-by-step explanation:

[tex] - 2 \sqrt{48} \\ = - 2 \sqrt{16 \times 3} \\ = - 2 \sqrt{16} \times \sqrt{3} \\ = - 2 \times 4\sqrt{3} \\ = - 8 \sqrt{3} [/tex]

Helppp

The half-life of Radium-226 is 1590 years. If a sample contains 500 mg, how many mg will remain after 1000 years?---------

Answers

Using the exponential decay equation, we can see that after 1000 years we will have  178.43 mg  of Ra₂₂₆

How much will remain after 1000 years?

The decay of a radioactive substance, such as Radium-226, can be modeled by the exponential decay equation:

N(t) = N₀ * (1/2)^(t / T)

where:

N(t) is the amount of the substance remaining after time t

N₀ is the initial amount of the substance

t is the time elapsed

T is the half-life of the substance

Given that the half-life of Radium-226 is 1590 years and the initial amount is 500 mg, we can plug in these values into the equation and solve for N(1000), which represents the amount remaining after 1000 years.

N(1000) = 500 * (1/2)^(1000 / 1590)

N(1000) ≈ 178.43 mg

Learn more about half life at:

https://brainly.com/question/11152793

#SPJ1

Other Questions
Which models the changes in the availability of food energy as energy moves through an ecosystem? The table represents the quadratic functions f(x) and g(x).x f(x) g(x)8 16 44 4 10 0 04 4 18 16 4What transformation of f(x) will produce g(x)? g of x equals one fourth times f of x g of x equals f of the quantity of one fourth times x end quantity g(x) = 4f(x) g(x) = f(4x) Determine the intercepts of the line.Do not round your answers.5x 9 = 8y 3Please help Helen asked him _______how much the house was.how much was the househow much was the house?He asked _______.I worked wherewhere worked where I workedRumia said she _______ to watch French films. likedhad likedis likingBev _______ Benjamin that he had a day off on Friday.saidasketoldHe said _______ a good idea.that it wasme it wasto I it was Can you help me with this? Concord Company started operations on January 1, 2015, and has used the FIFO method of inventory valuation since its inception. In 2021, it decides to switch to the average-cost method. You are provided with the following information. Net Income Retained Earnings (Ending Balance) Under FIFO Under Average-Cost Under FIFO 2015 $98,000 $88,100 $102,500 2016 68,100 63,600 161,100 2017 90,900 81,500 234,200 2018 108,100 118,200 336,600 2019 324,300 313,800 595,400 2020 303,200 308,200 742,200 (a) What is the beginning retained earnings balance at January 1, 2017, if Concord prepares comparative financial statements starting in 2017 1. A boy moves on a skateboard at a constant velocity of 3 m-s-'. Thecombined mass of the boy and the skateboard is 40 kg. He catches a bag offlour of mass 5 kg that is thrown to him horizontally at 6 m-s-!. Determinethe velocity of the boy after catching the bag of flour. (2 m-s ' in his originaldirection) When Melody first demonstrates the Medi-Talker's abilities to her generaleducation class, many are shocked because.... Calculate moles of al2(so4)3produced if you have 8.5 moles h2so4that are available to react. The movement of alleles in and out of a population caused by individuals or gametes is known as:. P5-5 Vocabulario 1: Una leccin de anatoma (Practice it!) PLEASE HELP IM BEGGINGthe net of the triangular prism and its appropriate dimensions are shown in the diagram which measurement is the closest to the total surface area of the triangular prism in square inches Pls answer this asap Its due tomorrow I really need help Please help me need this today and The book/ story is The Night Face Up1. Which images in this story linger in your mind? Explain. 2. Describe the motorcycle accident. Which symptoms resulting from the accident may affect the rider's perception of reality? 3. Why is the Moteca man running through the forest? Until the last paragraph, what does the narrator lead us to believe about the scenes involving the Moteca man? 4. How would you explain the title of the story? Do you think the title is effective? Explain. 5. What details does Cortzar use to convey the sensation of waking from a dream? how do these details contribute to the sense that perception and reality are often two different things? I need help on this one question in scienceThis image of a tiny fraction of the night sky was taken through a powerful telescope. Many of the objects seen in the image are galaxies similar to the Milky Way. Telescopes have taken many images like this one, but of different fractions of the night sky. What do these images suggest? A. Each galaxy contains an equal number of stars. B. The Milky Way is the largest galaxy in the Universe. C. There are no other galaxies in the Universe. D. There are many other galaxies in the Universe a single-turn current loop, carrying a current of 4.00 a, is in the shape of a right triangle with sides 50.0, 120, and 130 cm. the loop is in a uniform magnetic field of magnitude 75.0 mt whose direc- tion is parallel to the current in the 130 cm side of the loop. what is the magnitude of the magnetic force on (a) the 130 cm side, (b) the 50.0 cm side, and (c) the 120 cm side? (d) what is the magnitude of the net force on the loop? Write a balanced equation for the following reaction, which occurs in an acid solution. Fe + Cl2 ---> Fe3+ + Cl- El mastil de un velero se halla unido a la proa y a la popa por dos cables que forman con la cubierta, angulo de 45 grados y 60 grados, respectivamente. si el barco tiene una longitud de 25m, cual es la altura del mastil? Find the properties for the ellipse with the equation x^2/169 + y^2/144 = 1latus rectum =a. 288/13b. 328/12c. 288/12